Difference between revisions of "1967 IMO Problems/Problem 2"

m
m
 
Line 98: Line 98:
  
 
Given a tetrahedron <math>T</math>, and an edge <math>e_1</math> of it, we can find another
 
Given a tetrahedron <math>T</math>, and an edge <math>e_1</math> of it, we can find another
tetrahedron <math>U</math> such that <math>volume(U) > volume(T)</math>, with an edge
+
tetrahedron <math>U</math> such that <math>\mathbf{volume}(U) > \mathbf{volume}(T)</math>,
<math>f_1 > e_1</math>, and such that all the other edges of <math>U</math> are equal
+
with an edge <math>f_1 > e_1</math>, and such that all the other edges of <math>U</math>
to the corresponding edges of <math>T</math>, <math>\mathbf{unless}</math> the edge <math>e_1</math>
+
are equal to the corresponding edges of <math>T</math>, <math>\mathbf{unless}</math> the
stretches between sides of <math>T</math> which are perpendicular.
+
edge <math>e_1</math> stretches between sides of <math>T</math> which are perpendicular.
  
 
(By "stretches" I mean that the end points of the edge are the vertices
 
(By "stretches" I mean that the end points of the edge are the vertices
Line 110: Line 110:
 
edges <math>e_1, \dots, e_6</math>, such that <math>e_2, \dots, e_6 \le 1</math>.  We will
 
edges <math>e_1, \dots, e_6</math>, such that <math>e_2, \dots, e_6 \le 1</math>.  We will
 
show that if there is an edge <math>e_m < 1</math> among <math>e_2, \dots, e_6</math> then
 
show that if there is an edge <math>e_m < 1</math> among <math>e_2, \dots, e_6</math> then
there is a tetrahedron <math>T_1</math> with the same edges as <math>T</math>, except that
+
there is a tetrahedron <math>T_1</math> with bigger volume than the volume of <math>T</math>,
<math>e_m</math> is replaced by an edge of size <math>1</math>.
+
with the same edges as <math>T</math>, except that <math>e_m</math> is replaced by an edge of
 +
size <math>1</math>.
  
 
Case 1: If <math>T</math> does not have any sides which are perpendicular, then
 
Case 1: If <math>T</math> does not have any sides which are perpendicular, then
Line 120: Line 121:
 
then again, the existence of <math>T_1</math> follows from the corollary.  If
 
then again, the existence of <math>T_1</math> follows from the corollary.  If
 
<math>C_2D</math> is the only side <math>< 1</math>, then all the other sides are <math>= 1</math>, then
 
<math>C_2D</math> is the only side <math>< 1</math>, then all the other sides are <math>= 1</math>, then
<math>\triangle ABD</math>, \triangle AC_2B<math> are equilateral with sides </math>= 1<math>, and
+
<math>\triangle ABD, \triangle AC_2B</math> are equilateral with sides <math>= 1</math>, and
the planes can not be perpendicular since </math>C-2D < 1$.
+
the planes can not be perpendicular since <math>C_2D < 1</math>.
  
 
Case 3: Assume that three sides are perpendicular.
 
Case 3: Assume that three sides are perpendicular.
Line 127: Line 128:
 
[[File:Prob_1967_2_fig3.png|600px]]
 
[[File:Prob_1967_2_fig3.png|600px]]
  
 
+
Assume each pair of the planes meeting at <math>A</math> are perpendicular.  Since
 
+
at least two of the edges <math>BD, BC, CD \le 1</math> it follows that
 
+
<math>AB, AC, AD < 1</math> (the sides of the right angle in a right triangle are
 
+
less than the hypotenuse).  So we can apply the corollary, and find a
===Proof===
+
tetrahedron <math>T_1</math> with bigger volume than the volume of <math>T</math>, with sides
 +
equal to those of <math>T</math>, except that <math>AB</math> is replaced by a side <math>= 1</math>.
  
  

Latest revision as of 18:36, 14 September 2024

Prove that if one and only one edge of a tetrahedron is greater than $1$, then its volume is $\le \frac{1}{8}$.

Solution

Assume $CD>1$ and let $AB=x$. Let $P,Q,R$ be the feet of perpendicular from $C$ to $AB$ and $\triangle ABD$ and from $D$ to $AB$, respectively.

Suppose $BP>PA$. We have that $CP=\sqrt{CB^2-BT^2}\le\sqrt{1-\frac{x^2}4}$, $CQ\le CP\le\sqrt{1-\frac{x^2}4}$. We also have $DQ^2\le\sqrt{1-\frac{x^2}4}$. So the volume of the tetrahedron is $\frac13\left(\frac12\cdot AB\cdot DR\right)CQ\le\frac{x}6\left(1-\frac{x^2}4\right)$.

We want to prove that this value is at most $\frac18$, which is equivalent to $(1-x)(3-x-x^2)\ge0$. This is true because $0<x\le 1$.

The above solution was posted and copyrighted by jgnr. The original thread can be found here: [1]


Remarks (added by pf02, September 2024)

The solution above is essentially correct, and it is nice, but it is so sloppily written that it borders the incomprehensible. Below I will give an edited version of it for the sake of completeness.

Then, I will give a second solution to the problem.

A few notes which may be of interest.

The condition that one side is greater than $1$ is not really necessary. The statement is true even if all sides are $\le 1$. What we need is that no more than one side is $> 1$.

The upper limit of $1/8$ for the volume of the tetrahedron is actually reached. This will become clear from both solutions.


Solution

Assume $CD > 1$ and assume that all other sides are $\le 1$. Let $AB = x$. Let $P, Q, R$ be the feet of perpendiculars from $C$ to $AB$, from $C$ to the plane $ABD$, and from $D$ to $AB$, respectively.

Prob 1967 2 fig1.png

At least one of the segments $AP, PB$ has to be $\ge \frac{x}{2}$. Suppose $PB \ge \frac{x}{2}$. (If $AP$ were bigger that $\frac{x}{2}$ the argument would be the same.) We have that $CP = \sqrt{BC^2 - PB^2} \le \sqrt{1 - \frac{x^2}{4}}$. By the same argument in $\triangle ABD$ we have $DR \le \sqrt{1 - \frac{x^2}{4}}$. Since $CQ \perp$ plane $ABD$, we have $CQ \le CP$, so $CQ \le \sqrt{1 - \frac{x^2}{4}}$.

The volume of the tetrahedron is

$V = \frac{1}{3} \cdot ($area of $\triangle ABD) \cdot$(height from $C) = \frac{1}{3} \cdot \left( \frac{1}{2} \cdot AB \cdot DR \right) \cdot CQ \le \left( \frac{1}{6} \cdot x \cdot \sqrt{1 - \frac{x^2}{4}} \cdot \sqrt{1 - \frac{x^2}{4}} \right) = \frac{x}{6} \left( 1 - \frac{x^2}{4} \right)$.

We need to prove that $\frac{x}{6} \left( 1 - \frac{x^2}{4} \right) \le \frac{1}{8}$. Some simple computations show that this is the same as $(1 - x)(3 - x - x^2) \ge 0$. This is true because $0 < x \le 1$, and $-x^2 - x + 3 \ge 0$ on this interval.

Note: $V = \frac{1}{8}$ is achieved when $x = 1$ and all inequalities are equalities. This is the case when all sides except $AD$ are $= 1$, $P, R$ are midpoints of $AB$ and $Q = P$ (in which case the planes $ABC, ABD$ are perpendicular). In this case, $AD = \frac{\sqrt{6}}{2}$, as can be seen from an easy computation.


Solution 2

We begin with two simple propositions.

Proposition

Let $ABCD$ be a tetrahedron, and consider the transformations which rotate $\triangle ABC$ around $AB$ while keeping $\triangle ABD$ fixed. We get a set of tetrahedrons, two of which, $ABC_1D$ and $ABC_2D$ are shown in the picture below. The lengths of all sides except $CD$ are constant through this transformation.

Prob 1967 2 fig2.png

1. Assume that the angles between the planes $ABD$ and $ABC$, and $ABD$ and $ABC_1$ are both acute. If the perpendicular from $C_1$ to the plane $ABD$ is larger that the perpendicular from $C$ to the plane $ABD$ then the volume of $ABC_1D$ is larger than the volume of $ABCD$.

2. Furthermore, the tetrahedron $ABC_2D$ obtained when the position of $C_2$ is such that the planes $ABD$ and $ABC_2$ are perpendicular has the maximum volume of all tetrahedrons obtained from rotating $\triangle ABC$ around $AB$.

These statements are intuitively clear, since the volume $V$ of the tetrahedron $ABCD$ is given by

$V = \frac{1}{3} \cdot ($area of $\triangle ABD) \cdot ($height from $C)$.

A formal proof is very easy, and I will skip it.

Corollary

Given a tetrahedron $T$, and an edge $e_1$ of it, we can find another tetrahedron $U$ such that $\mathbf{volume}(U) > \mathbf{volume}(T)$, with an edge $f_1 > e_1$, and such that all the other edges of $U$ are equal to the corresponding edges of $T$, $\mathbf{unless}$ the edge $e_1$ stretches between sides of $T$ which are perpendicular.

(By "stretches" I mean that the end points of the edge are the vertices on the two sides which are not common to the two sides. In the picture above, $DC_2$ stretches between the sides $ABD, AC_2B$ of $ABC_2D$.)

Now for the proof of the problem, assume we have a tetrahedron $T$ with edges $e_1, \dots, e_6$, such that $e_2, \dots, e_6 \le 1$. We will show that if there is an edge $e_m < 1$ among $e_2, \dots, e_6$ then there is a tetrahedron $T_1$ with bigger volume than the volume of $T$, with the same edges as $T$, except that $e_m$ is replaced by an edge of size $1$.

Case 1: If $T$ does not have any sides which are perpendicular, then the existence of $T_1$ follows from the corollary.

Case 2: Assume $T$ has two sides which are perpendicular (like $ABC_2D$ in the picture above). If any of the sides other than $C_2D$ are $< 1$, then again, the existence of $T_1$ follows from the corollary. If $C_2D$ is the only side $< 1$, then all the other sides are $= 1$, then $\triangle ABD, \triangle AC_2B$ are equilateral with sides $= 1$, and the planes can not be perpendicular since $C_2D < 1$.

Case 3: Assume that three sides are perpendicular.

Prob 1967 2 fig3.png

Assume each pair of the planes meeting at $A$ are perpendicular. Since at least two of the edges $BD, BC, CD \le 1$ it follows that $AB, AC, AD < 1$ (the sides of the right angle in a right triangle are less than the hypotenuse). So we can apply the corollary, and find a tetrahedron $T_1$ with bigger volume than the volume of $T$, with sides equal to those of $T$, except that $AB$ is replaced by a side $= 1$.




(Solution by pf02, September 2024)

TO BE CONTINUED. DOING A SAVE MIDWAY SO I DON'T LOOSE WORK DONE SO FAR.


See Also

1967 IMO (Problems) • Resources
Preceded by
Problem 1
1 2 3 4 5 6 Followed by
Problem 3
All IMO Problems and Solutions